Bohmsches Schlupfloch in PBR-ähnlichen Theoremen

Ich greife diese Frage wieder auf und erweitere sie, wegen der neuen Veröffentlichung heute von Aaronson et al . Die allgemeinere Frage lautet:

Wie verhält sich die Quantenpotenzial-Bohmsche Mechanik zu No-Go-Theoremen für psi-epistemische Theorien?

Nach der Bohmschen Mechanik gibt es wie in der konventionellen Quantenmechanik eine Wellenfunktion, die sich nach einer Schrödinger-Gleichung entwickelt, und dann einen Satz klassischer Freiheitsgrade, die sich entsprechend dem Gradienten der Phase dieser Wellenfunktion entwickeln. Eine Born-Regel-Wahrscheinlichkeitsverteilung für die klassischen Freiheitsgrade wird durch dieses Bewegungsgesetz bewahrt, sodass die Bohmsche Mechanik eine deterministische Theorie der verborgenen Variablen liefert, wenn auch eine nichtlokale.

Die Bewegungsgleichungen für die klassischen Variablen können jedoch in Bezug auf zwei Potentiale umgeschrieben werden, das lokale Potential, das in der Schrödinger-Gleichung erscheint, und ein nichtlokales "Quantenpotential", das von der Form der Wellenfunktion abhängt. Das bedeutet, dass die Bohmsche Mechanik für jede gegebene Wellenfunktion durch eine nichtlokale deterministische Theorie ersetzt werden kann, in der es keine Wellenfunktion gibt. (Beachten Sie, dass das Quantenpotential zeitabhängig ist, es sei denn, die Wellenfunktion war ein Energieeigenzustand; danke an Tatiana Seletskaia für die Betonung dieses Punktes.)

In der Zwischenzeit besteht in letzter Zeit ein theoretisches Interesse daran, sogenannte "psi-epistemische" Theorien versteckter Variablen auszuschließen, in denen die Quantenwellenfunktion nicht Teil der Ontologie ist. Offensichtlich ist die „Quantenpotenzial-Bohmsche Mechanik“ eine psi-epistemische Theorie. Es sollte für dieses Unternehmen relevant sein. Aber es kann einige mentale Gymnastik erfordern, um es mit den PBR-ähnlichen Theoremen in Kontakt zu bringen, denn die Abbildung von der Quantenmechanik auf diese Quantenpotentialtheorien ist eine Eins-zu-Viele: Szenarien mit denselben Quantenbewegungsgleichungen, aber unterschiedlichen Anfangswerten Bedingungen für die Wellenfunktion, entsprechen verschiedenen Bewegungsgleichungen in einer Quantenpotentialtheorie.

ANMERKUNG 1: Empirisch müssen wir nur eine Welt erklären, also sollte ein Theoretiker des realen Quantenpotentials im Prinzip nicht einmal seinen Formalismus benötigen, um mit der Gesamtheit einer Quantentheorie übereinzustimmen. Die Quantenkosmologie benötigt nur eine Wellenfunktion des Universums, und so müsste ein solcher Theoretiker nur ein "kosmisches Quantenpotential" berücksichtigen, um seine Theorie zu definieren.

ANMERKUNG 2: Hier ist die Originalversion dieser Frage, die ich im Januar gestellt habe:

Das „PBR-Theorem“ ( Pusey-Barrett-Rudolph ) soll zeigen, dass man die Vorhersagen der Quantenmechanik nicht reproduzieren kann, ohne anzunehmen, dass Wellenfunktionen real sind. Aber es schien mir immer offensichtlich, dass dies falsch war, weil man die Bohmsche Mechanik so umschreiben kann, dass es keine "Pilotwelle" gibt - man muss nur die Bewegungsgleichung für die Bohmschen Teilchen umschreiben, so dass der von der Pilotwelle kommende Einfluss reproduziert wird ein nichtlokales Potential . Kann jemand erklären, warum der Abzug von PBR diese Möglichkeit übersieht?

Ich glaube immer noch, dass meine Antwort - dass die Bohmsche Mechanik psi-ontisch ist - die richtige ist, obwohl ich die Aaronson et al. Papier noch. Ich habe meine Antwort aktualisiert, um Ihre aktualisierte Frage zu beantworten. (Das Neue steht im letzten Absatz.)
Ich denke, eine allgemeinere Art, diese Frage auszudrücken, wäre: "Kann die Bohmsche Mechanik als psi-epistemische Theorie ausgedrückt werden?" Ich vermute, die Antwort darauf wäre nein, aber ich weiß es nicht genau. (Aber ich glaube nicht, dass es auf dieser Seite noch viele Benutzer gibt, deren Interessen in diesem Bereich liegen.)
Scott Aaronson hat mir gesagt, 1) dass eine andere solche Darstellung existiert und hier diskutiert wurde physical.stackexchange.com/questions/38064/… ...
... 2) dass es bei PBR-Theoremen um Theorien geht, bei denen es eine Überlappung in der ontischen Basis der epistemischen Zustände gibt, dh wenn psi1 und psi2 nicht-orthogonale Quantenzustände sind, dann überlappen sich die entsprechenden Sätze möglicher ontischer Zustände. Hoffentlich können wir in ein paar Tagen all diese Teile zusammenfügen und eine umfassende Antwort finden.

Antworten (3)

Ich habe einmal ein Online-Seminar von Robert Spekkens gesehen, der etwas in der Art gesagt hat, dass No-Go-Theoreme interessant sind, weil sie einschränken, wie eine epistemische Interpretation der Quantenmechanik aussehen kann. Jedes No-Go-Theorem macht eine Reihe von Annahmen, und wenn das Theorem richtig ist, wissen wir, dass wir mindestens eine dieser Annahmen vermeiden müssen, wenn wir eine erfolgreiche Theorie aufstellen wollen.

Das Pusey-Barrett-Rudolph-Papier formuliert einige seiner Annahmen. (Sie tun dies am deutlichsten in den abschließenden Abschnitten.) Es kann durchaus zusätzliche nicht erwähnte Annahmen geben (z. B. Kausalität), aber diejenigen, die sie ausdrücklich erwähnen, sind:

  • Es gibt einen objektiven körperlichen Zustand λ für jedes Quantensystem

  • Es gibt welche λ ' die zwischen einem Paar unterschiedlicher Quantenzustände geteilt werden kann ψ 1 Und ψ 2 . Das ist, P ( λ = λ ' | ψ = ψ 1 ) Und P ( λ = λ ' | ψ = ψ 2 ) sind beide nicht Null. (Das bedeutet nach Spekkens' Definition, dass eine Interpretation epistemisch ist.)

  • Die Ergebnisse der Messungen hängen nur von ab λ und die Einstellungen des Messgeräts (obwohl es auch Stochastik geben kann)

  • Räumlich getrennte Systeme, die unabhängig voneinander hergestellt wurden, haben separate und unabhängige λ 'S.

Daraus leiten sie einen Widerspruch ab. Jede Theorie, die nicht alle diese Annahmen trifft, wird von ihrem Ergebnis nicht beeinflusst.

Ich bin mir ziemlich sicher, dass unter der Standardformulierung der Bohmschen Mechanik die zweite verstößt, dh die Bohmsche Mechanik ist keine Erkenntnistheorie im Sinne des Spekkens. Dies liegt daran, dass in der Bohmschen Mechanik der physikalische Zustand λ besteht sowohl aus der realen Position des Teilchens als auch aus dem "Quantenpotential", und letzteres steht in einer Eins-zu-Eins-Beziehung zum Quantenzustand.

Wenn ich das richtig verstehe, schlagen Sie vor, dass Sie stattdessen an den physischen Zustand denken könnten, λ , da sie nur aus den Positionen und Geschwindigkeiten der Teilchen besteht, wobei das nichtlokale Potential stattdessen als Teil der Bewegungsgleichungen betrachtet wird. Aber in diesem Fall wird die dritte obige Annahme verletzt, weil Sie zusätzlich noch das Potenzial kennen müssen λ , um die Messergebnisse vorherzusagen. Da diese Formation gegen die Annahmen des Arguments von Pusey, Barrett und Rudolph verstoßen würde, würde ihr Ergebnis darauf nicht zutreffen.

In Ihrer aktualisierten Frage stellen Sie klar, dass Ihr Vorschlag darin besteht, die Wellenfunktion auf einen bestimmten Wert festzulegen. In diesem Fall ist es sicherlich richtig, dass sich die Bohmsche Mechanik auf Teilchen reduziert, die sich gemäß deterministischen, aber nicht lokalen Bewegungsgleichungen bewegen. Aber dann hat man nur noch ein Teilmodell der Quantenmechanik, weil man nichts mehr darüber sagen kann, was passiert, wenn man die Wellenfunktion verändert. Mein starker Verdacht ist, dass Sie bei diesem Ansatz am Ende ein epistemisches Modell erhalten werden, aber es wird nur ein Modell einer begrenzten Teilmenge der Quantenmechanik sein, und dies wird dazu führen, dass Pusey et al . Das Ergebnis von ist nicht anwendbar.

In der üblichen Formulierung der Bohmschen Mechanik haben die Bewegungsgleichungen für die Teilchen eine gewisse Abhängigkeit von einer Wellenfunktion, die als "Pilotwelle" wirkt. Aber ich sage, Sie können einfach eine gegebene Wellenfunktion in diese Bewegungsgleichungen einsetzen, so dass Sie nichts als Teilchen übrig haben, die gemäß einem nichtlokalen Potential interagieren. In dieser veränderten Formulierung ist das Quantenpotential also nicht Teil des physikalischen Zustands, sondern Teil der Bewegungsgleichungen.
Nun gut, aber in diesem Fall müssen Sie, wenn ich Sie richtig verstehe, um die Ergebnisse von Messungen vorherzusagen, immer noch die Wellenfunktion kennen, sonst hätten Sie nur einige teilweise unbekannte Bewegungsgleichungen. Wenn Sie sich also dafür entscheiden, die Wellenfunktion eines Teils nicht zu zählen λ , verstößt die Theorie gegen die dritte Annahme in meiner Liste, daher gilt das Pusey-Barrett-Rudolph-Ergebnis nicht dafür.
Ich habe meinen vorherigen Kommentar in die Antwort bearbeitet.

Mathematisch ausgedrückt zeigt das PBR-Theorem, dass die Beziehung zwischen den Zuständen einer grundlegenderen Theorie und den Zuständen der Quantenmechanik (Wellenfunktionen) funktional ist (dh zu jedem Zustand der grundlegenderen Theorie gehört höchstens eine Wellenfunktion). ; die Beziehung muss natürlich nicht auf oder eins-zu-eins sein).

Wenn dies ausreicht, um zu sagen, "Wellenfunktionen sind real" (Ihr Zitat), dann sei es so. Im Allgemeinen lässt sich dieser Schluss aber aus dem im ersten Absatz genannten Ergebnis nicht (durch Argumente) „erlangen“. Deshalb verwendet man eine Sprache (psi-epistemisch usw.), die genau die oben erwähnte funktionale Abhängigkeit kodifiziert. Meiner Ansicht nach ist es ein Fehler, den Ausschluss einer nichtfunktionalen Beziehung zwischen den Zuständen einer grundlegenderen Theorie und Wellenfunktionen als Zeichen dafür zu nehmen, dass "Wellenfunktionen real sind".

Lassen Sie mich abschließend noch anmerken, dass ein Zustand in der Bohmschen Mechanik aus dem Paar (Wellenfunktion, Teilchenpositionen) besteht. Daher ist die Beziehung zwischen den Zuständen in dieser Theorie und den Zuständen in der Quantenmechanik sicherlich funktional (Projektion auf die erste Komponente, wenn Sie möchten). - Wie erwartet kein Problem und keine Auswirkungen hier.

Beste, JK

Ps: Bitte weisen Sie darauf hin, wenn hier ein Fehler vorliegt! Pps: Auf die obige Weise ausgedrückt ist das PBR-Theorem fast "trivial wahr".

Leider habe ich nicht die Punkte, um Kommentare zu anderen Antworten zu hinterlassen, da Nathaniel Recht hat, aber da keine Antwort akzeptiert wurde, ist hier die Antwort auf Ihre Frage mit einem Zitat, um sie zu untermauern:

Sie haben eine falsche Annahme getroffen.

In der Zwischenzeit besteht in letzter Zeit ein theoretisches Interesse daran, sogenannte "psi-epistemische" Theorien versteckter Variablen auszuschließen, in denen die Quantenwellenfunktion nicht Teil der Ontologie ist. Offensichtlich ist die „Quantenpotenzial-Bohmsche Mechanik“ eine psi-epistemische Theorie. Es sollte für dieses Unternehmen relevant sein.

Die Bohmsche Mechanik ist nicht ψ-epistemisch, sie ist ψ-ergänzt (eine Teilmenge der ψ-Ontik). Siehe http://dx.doi.org/10.1007/s10701-009-9347-0

Ich spreche von einer Neufassung der Bohmschen Mechanik, bei der die Wellenfunktion in die Bewegungsgleichung für die konfigurativen Beables aufgenommen wird. In einer solchen Theorie existiert ψ nicht mehr.
Also wie die Formulierung von Poirier ( dx.doi.org/10.1016/j.chemphys.2009.12.024 )? Da die Trajektorien immer noch real und nicht überlappend sind, ist jeder physikalische Zustand immer noch konsistent mit nur einem reinen Quantenzustand. Daher kann es per Definition nicht als ψ-epistemisch betrachtet werden (wie oben von Harrigan und Spekkens definiert).
Das scheint eine Version von "vielen interagierenden Welten" zu sein, in der alle Bohmschen Trajektorien für ein gegebenes ψ auf einmal real sind. Ich spreche nur von einer Trajektorie mit einer Bewegungsgleichung, in der die Rolle von ψ vollständig durch einen Potentialterm ersetzt wird.
Dies ist ein weit verbreitetes Missverständnis. In BM wird das Führungsfeld über den 3N-Raum als real betrachtet (im Fall von spinlosen nichtrelativistischen Teilchen). Aus Bell (1989) : „Niemand kann diese Theorie verstehen, bis er bereit ist, darüber nachzudenken Ψ als reales objektives Feld und nicht nur als "Wahrscheinlichkeitsamplitude". Obwohl es sich nicht im 3-Raum, sondern im 3N-Raum ausbreitet. Sie scheinen eine einzigartige Interpretation schaffen zu wollen, wie von Norsen (2016) vorgeschlagen . Das wäre aber kein BM.
Ich spreche nicht von einer Ein-Teilchen-Flugbahn. Sie können Ihre N Teilchen haben, der Punkt ist, dass Sie, egal mit welchem ​​ψ über dem 3N-Raum Sie beginnen, es in die Bohmsche Bewegungsgleichung für die N Teilchen einsetzen können und eine neue globale Bewegungsgleichung mit einem bestimmten nichtlokalen Potential erhalten (Norsens Q).